- Tue May 15, 2018 4:38 pm
#45694
Complete Question Explanation
(The complete setup for this game can be found here: lsat/viewtopic.php?t=16967)
The correct answer choice is (D)
If W and Z are both selected, then H must also be selected, and F, M, and U all cannot be selected.
Answer choice (A) is incorrect because U cannot be selected.
Answer choice (B) is incorrect because K and L cannot both be selected (two students).
Answer choice (C) is incorrect because M cannot be selected.
Answer choice (D) is the correct answer choice.
Answer choice (E) is incorrect because F cannot be selected.
(The complete setup for this game can be found here: lsat/viewtopic.php?t=16967)
The correct answer choice is (D)
If W and Z are both selected, then H must also be selected, and F, M, and U all cannot be selected.
Answer choice (A) is incorrect because U cannot be selected.
Answer choice (B) is incorrect because K and L cannot both be selected (two students).
Answer choice (C) is incorrect because M cannot be selected.
Answer choice (D) is the correct answer choice.
Answer choice (E) is incorrect because F cannot be selected.